LSAT and Law School Admissions Forum

Get expert LSAT preparation and law school admissions advice from PowerScore Test Preparation.

User avatar
 Dave Killoran
PowerScore Staff
  • PowerScore Staff
  • Posts: 5853
  • Joined: Mar 25, 2011
|
#33981
Must Be True - SN. The correct answer choice is (E).

This is a classic Conditional Reasoning and Must Be True combination problem. Let's start by breaking down the stimulus:

  • The first sentence provides background information about efficiency and redundancy: they are contradictory but can be used together in communications.

    The next two sentences present conditional statements:

    (2): CE :arrow: EPU
    (Completely efficient :arrow: every permutation understandable)


    (3) IR :arrow: EPU
    (Imperfect receptor :arrow: every permutation not understandable)
As you may recall, when two separate sufficient conditions each have opposite necessary conditions, the two sufficient conditions cannot co-exist:


..... ..... C :dblline: IR


This occurs because each sufficient condition relies on a necessary condition, and those two necessary conditions are opposite of each other (and thus cannot exist simultaneously). Imagine if the two statements were instead: To go to dinner we must be rich and to go to the movies we must be poor. If those two statements are true, then no one can go to both a dinner and a movie (and we'll ignore the time sequence aspect here and assume no one loses their fortune in between dinner and a movie :-D ).

Given the above analysis, we can attack the answer choices:

Answer choice (A): While this answer might make some sense in the real world, there is no justification for the statement base don what is in the stimulus.

Answer choice (B): While the two are mutually exclusive and cannot both be achieved, there is no indication from the stimulus that one of them can't be achieved.

Answer choice (C): Again, there is no evidence to support this statement.

Answer choice (D): This is a Mistaken Negation of the statement in the third sentence, and is thus incorrect.

Answer choice (E): This is the correct answer. This reflects the inference drawn from combining the second and third sentences, and is a variation on the expected answer (that CE and IR cannot coexist).
 Trey
  • Posts: 3
  • Joined: Dec 21, 2018
|
#61345
Why is E a better answer choice than D?
User avatar
 Dave Killoran
PowerScore Staff
  • PowerScore Staff
  • Posts: 5853
  • Joined: Mar 25, 2011
|
#61363
Trey wrote:Why is E a better answer choice than D?
Hi Trey, take a look at the above post now, I've just posted in an official explanation :)

Get the most out of your LSAT Prep Plus subscription.

Analyze and track your performance with our Testing and Analytics Package.